Difference between revisions of "2009 AMC 8 Problems/Problem 19"

(Problems)
Line 1: Line 1:
==Problems==
+
==Problem==
 
Two angles of an isosceles triangle measure <math> 70^\circ</math> and <math> x^\circ</math>. What is the sum of the three possible values of <math> x</math>?
 
Two angles of an isosceles triangle measure <math> 70^\circ</math> and <math> x^\circ</math>. What is the sum of the three possible values of <math> x</math>?
  
Line 7: Line 7:
 
\textbf{(D)}\    165  \qquad
 
\textbf{(D)}\    165  \qquad
 
\textbf{(E)}\    180</math>
 
\textbf{(E)}\    180</math>
 +
 +
 +
==Solution==
 +
 +
There are 3 cases: where <math> x^\circ</math> is a base angle with the <math> 70^\circ</math> as the other angle, where <math> x^\circ</math> is a base angle with <math> 70^\circ</math> as the vertex angle, and where <math> x^\circ</math> is the vertex angle with <math> 70^\circ</math> as a base angle.
 +
 +
Case 1: <math> x^\circ</math> is a base angle with the <math> 70^\circ</math> as the other angle:
 +
Here, <math> x=70</math>, since base angles are congruent.
 +
 +
Case 2: <math> x^\circ</math> is a base angle with <math> 70^\circ</math> as the vertex angle:
 +
Here, the 2 base angles are both <math> x^\circ</math>, so we can use the equation <math> 2x+70=180</math>, which simplifies to <math> x=55</math>.
 +
 +
Case 3: <math> x^\circ</math> is the vertex angle with <math> 70^\circ</math> as a base angle:
 +
Here, both base angles are <math> 70^\circ</math>, since base angles are congruent. Thus, we can use the equation <math> x+140=180</math>, which simplifies to <math> x=40</math>.
 +
 +
Adding up all the cases, we get <math>70+55+40=165</math>, so the answer is <math> \textbf{(D)}\    165</math>.
  
 
==See Also==
 
==See Also==
 
{{AMC8 box|year=2009|num-b=18|num-a=20}}
 
{{AMC8 box|year=2009|num-b=18|num-a=20}}

Revision as of 18:17, 12 November 2012

Problem

Two angles of an isosceles triangle measure $70^\circ$ and $x^\circ$. What is the sum of the three possible values of $x$?

$\textbf{(A)}\   95    \qquad \textbf{(B)}\    125   \qquad \textbf{(C)}\    140   \qquad \textbf{(D)}\    165   \qquad \textbf{(E)}\     180$


Solution

There are 3 cases: where $x^\circ$ is a base angle with the $70^\circ$ as the other angle, where $x^\circ$ is a base angle with $70^\circ$ as the vertex angle, and where $x^\circ$ is the vertex angle with $70^\circ$ as a base angle.

Case 1: $x^\circ$ is a base angle with the $70^\circ$ as the other angle: Here, $x=70$, since base angles are congruent.

Case 2: $x^\circ$ is a base angle with $70^\circ$ as the vertex angle: Here, the 2 base angles are both $x^\circ$, so we can use the equation $2x+70=180$, which simplifies to $x=55$.

Case 3: $x^\circ$ is the vertex angle with $70^\circ$ as a base angle: Here, both base angles are $70^\circ$, since base angles are congruent. Thus, we can use the equation $x+140=180$, which simplifies to $x=40$.

Adding up all the cases, we get $70+55+40=165$, so the answer is $\textbf{(D)}\    165$.

See Also

2009 AMC 8 (ProblemsAnswer KeyResources)
Preceded by
Problem 18
Followed by
Problem 20
1 2 3 4 5 6 7 8 9 10 11 12 13 14 15 16 17 18 19 20 21 22 23 24 25
All AJHSME/AMC 8 Problems and Solutions